Pharmacology-- Curative

Réussis tes devoirs et examens dès maintenant avec Quizwiz!

Ipratropium and Albuterol

A client with COPD may experience acute bronchospasm that can be life-threatening. The medication of choice for this occurrence is albuterol, a short-acting β2 agonist. Another medication effective in the prevention and reduction of acute bronchospasm is ipratropium. This medication is a cholinergic antagonist that works by suppressing the parasympathetic nervous system so that the sympathetic nervous system can release norepinephrine to relieve and prevent bronchoconstriction. Ipratropium is considered a maintenance medication to prevent bronchoconstriction and can be taken after albuterol in the event of unrelieved symptoms. The nurse instructs a client with chronic obstructive pulmonary disease (COPD) about ipratropium and albuterol. Which client statement indicates that additional teaching is required? "Ipratropium is used first when I cannot catch my breath." Ipratropium is a cholinergic antagonist that can stop or reduce the effects of a life-threatening bronchospasm. It can be used as a reliever medication, but it is not the first choice. NOT: "Taking these medications together relieves chest tightness." In the event of an acute bronchospasm, albuterol should be used first, followed by ipratropium bromide if symptoms are not relieved. "Ipratropium is taken every day on a schedule." Ipratropium is a cholinergic antagonist that is used as a maintenance medication in the treatment of COPD. "Albuterol works faster than ipratropium." Albuterol is a short-acting β2 agonist that relieves bronchospasms.

Vesicant chemotherapeutic agent

A vesicant is an agent that causes tissue irritation and possible tissue necrosis. Signs of extravasation (pain around the infusion site) should be taken seriously because of the risk of tissue necrosis. If extravasation of a vesicant is suspected, immediate actions include the following: 1. Stop the infusion. 2. Contact the healthcare provider. 3. Leave the IV catheter in place, and aspirate residual drug or blood from the catheter. Other actions taken will depend on the specific vesicant, so the nurse should consult facility guidelines. Do not massage or otherwise apply pressure to the site, as that could distribute vesicant into tissues. A client with a vesicant chemotherapeutic medication infusing into a peripheral IV access reports burning. The nurse immediately takes which of the following actions? Select All That Apply Contacts the healthcare provider The healthcare provider needs to be contacted immediately. Stops the medication infusion Additional vesicant coming into contact with tissues will cause further damage and must be avoided. Aspirates any remaining medication Aspiration is often recommended if possible, as it can remove additional solution still at the site. NOT: Removes the IV catheter The catheter should remain in place; the client may require immediate antidote therapy at the site of infusion. Provides gentle massage to the site Pressure on the area is not recommended and could further distribute the vesicant into tissues.

Activated charcoal

Activated charcoal is an antidote or absorbent that works by binding to toxic substances and inhibiting the absorption of these substances in the gastrointestinal tract. If a client is given a high dose too rapidly, nausea and vomiting can occur. Activated charcoal neither competes for drug receptor sites nor neutralizes all gastric contents. Binds with the drug Activated charcoal is classified as an antidote or adsorbent that works by binding to toxic substances, thus hindering their absorption in the gastrointestinal tract. NOT: A client is treated for an overdose of imipramine. Which action does the nurse expect when administering activated charcoal to this client? Induces vomiting Activated charcoal does not induce vomiting, but vomiting can occur if a high dose is ingested too rapidly. Competes for receptor sites Activated charcoal does not compete for drug receptor sites. Neutralizes all gastric contents Activated charcoal does not neutralize gastric contents.

Acyclovir (Zovirax)

Acyclovir is an antiviral medication used to treat herpes simplex virus. It interferes with the DNA synthesis of the virus and inhibits virus replication. Headache is identified as a common adverse effect of this medication. Because acyclovir does not affect the respiratory, sensory, or cardiovascular system, cough, tinnitus, and dysrhythmia do not occur as adverse effects. A client who is immunocompromised receives IV acyclovir as treatment for herpes zoster. Which assessment finding does the nurse identify as an adverse effect of this medication? Headache Headache is identified as a common adverse effect of this medication. NOT: Cough Cough is not identified as an adverse effect of this medication. Dysrhythmia Dysrhythmia is not identified as an adverse effect of this medication. Tinnitus Tinnitus is not identified as an adverse effect of this medication.

Alendronate (Fosamax)

Alendronate is a bone metabolism regulator used to treat osteoporosis. The most common side effects occur to the gastrointestinal tract and include nausea and dyspepsia. Side effects can also occur to the musculoskeletal system and include the development of myalgias. Confusion and hypertension are not side effects of alendronate. A client with osteoporosis is prescribed alendronate. For which of the following common side effects does the nurse monitor this client? Select All That Apply Dyspepsia Alendronate is a bone metabolism regulator used to treat osteoporosis. Dyspepsia is a common side effect of alendronate. Myalgia Alendronate is a bone metabolism regulator used to treat osteoporosis. Myalgia (muscle pain) is a common side effect of alendronate. Nausea Alendronate is a bone metabolism regulator used to treat osteoporosis. Nausea is a common side effect of alendronate. NOT: Confusion Confusion is not identified as a side effect of alendronate. Hypertension Hypertension is not identified as a side effect of alendronate.

Allopurinol (Zyloprim)

Allopurinol is an antigout medication that inhibits an enzyme that converts xanthine to uric acid. Alcohol should be avoided when taking allopurinol because it may inhibit the renal excretion of uric acid. A low-sodium diet is recommended to prevent the additional development of uric acid. A diet that promotes the development of uric acid should be avoided. This includes foods that contain purine, such as red meat. Purines are broken down into uric acid, which can cause an acute attack of gout. Allopurinol should be taken immediately after meals for the best tolerance, as 80%-90% of the medication is absorbed from the gastrointestinal tract. Fluid intake should be increased to at least 3 L per day or enough fluid to produce 2 L of urine. This will prevent the buildup of uric acid in the kidneys and reduce the risk of developing uric acid renal calculi. A client with gout is prescribed allopurinol. Which of the following statements does the nurse include when teaching the client about this medication? Select All That Apply "Avoid alcohol consumption." Allopurinol, an antigout medication, inhibits an enzyme that converts xanthine to uric acid. Alcohol may inhibit renal excretion of uric acid and should be avoided when taking allopurinol. "Increase your daily fluid intake to 3 liters." Fluid intake should be at least 3 L per day or sufficient enough to produce at least 2 L per day of urine. This prevents the buildup of uric acid in the kidneys and reduces the risk of uric acid renal calculi. "Take the medication after meals." The medication should be taken after meals for best tolerance. The tablet can be crushed and mixed with food. NOT: "Increase your intake of salt." A diet low in sodium should be consumed to prevent the development of uric acid. "Increase your intake of red meat." Red meat is high in purines, which are broken down into uric acid and can cause an acute attack of gout. Allopurinol is an antigout medication used primarily for prevention. It is not effective for treating an acute exacerbation because it takes 2-6 weeks to be effective. It blocks the enzyme xanthine oxidase, thereby preventing uric acid production. The client should also incorporate lifestyle changes to reduce uric acid production and associated complications of gout while taking allopurinol. Red meat should be avoided, as it is high in purines, which the body converts to uric acid. A client has been prescribed allopurinol. Which of the following instructions does the nurse include in client teaching? Select All That Apply "Increase your fluid intake to 3 liters per day." To help clear uric acid from the renal tubules and prevent renal calculi, increasing fluid intake is recommended unless contraindicated for the client. "Take allopurinol with or after meals." Allopurinol is given with meals to minimize gastrointestinal symptoms such as nausea, vomiting, and anorexia. "Avoid or reduce the use of alcohol." Alcohol increases uric acid production in the body and should be avoided. "Avoid taking aspirin unless directed." Aspirin has been shown to increase uric acid levels in some people, so the client should ask the healthcare provider before taking any over-the-counter aspirin products. NOT: "Increase your intake of meat rich in protein and iron." Red meat, especially organ meats, are high in purines, which the body converts to uric acid. Such meats should be avoided.

Aspirin (ASA)

Although aspirin is used for body temperature regulation, pain control, and inflammation reduction, for a client recovering from an acute MI, aspirin is prescribed to reduce platelet aggregation. This means that the platelets will not clump together and create a thrombus in the circulation. Aspirin makes the platelets less "sticky," so they flow freely through the bloodstream. Even though aspirin facilitates the flow of blood, it does not directly affect the dilation of coronary vessels. Nitrates are one category of medication that promote coronary vasodilation and may be prescribed for the client recovering from an acute MI or to reduce anginal pain. Aspirin does not affect clotting factors, but heparin does and may be prescribed for the client recovering from an acute MI. Aspirin has no effect on vitamin K synthesis. The anticoagulant warfarin blocks vitamin K synthesis, reducing the risk for blood clot development. A client with an acute myocardial infarction (MI) is prescribed 325 mg of aspirin by mouth. Which reason does the nurse give to the client as the reason for taking this medication? Decreases platelet aggregation Aspirin is a cyclooxygenase inhibitor that interferes with platelet aggregation. This means the platelets are less "sticky" and may decrease the risk of thrombus formation. NOT: Promotes coronary vasodilation Nitrates promote coronary vasodilation and may also be used to treat an acute MI or angina. Inactivates clotting factors Heparin inactivates clotting factors and may also be used to treat an acute MI. Blocks vitamin K synthesis Warfarin blocks vitamin K synthesis, decreasing the risk for blood clots. Warfarin is commonly used to treat deep vein thrombosis, pulmonary embolism, and atrial fibrillation.

Amlodipine

Amlodipine, a calcium channel blocker used for hypertension or stable angina, may cause side effects of palpitations, bradycardia, tachycardia, nausea, constipation, peripheral edema, flushing, and dyspnea. Instruct the client to notify the healthcare provider if any of these side effects or adverse effects occur with amlodipine. The healthcare provider prescribes amlodipine for a client with hypertension. The nurse teaches the client about which of the following reactions to amlodipine? All true: Constipation Clients taking amlodipine may experience nausea and constipation. Edema Clients taking amlodipine may experience peripheral edema. Flushing Clients taking amlodipine may experience flushing of the face, neck, and arms. Palpitations Clients taking amlodipine may experience palpitations, hypotension, bradycardia, or tachycardia. Shortness of breath Clients taking amlodipine may experience dyspnea

Ampicillin

Ampicillin is a broad-spectrum antibiotic that causes death of bacteria by inhibiting the cell wall synthesis of the organism. Shortness of breath must be reported without delay, as this may be the result of a true anaphylaxis reaction to the drug. An adverse effect of ampicillin is a rash, which could indicate a hypersensitivity to the medication. Because of this, a rash should be reported to the healthcare provider. A sudden-onset headache and elevated blood pressure are rarely symptoms of true anaphylaxis to penicillins, but they should be investigated. A client with a sinus infection is prescribed ampicillin. Which adverse effect does the nurse instruct the client to report immediately? Sudden-onset dyspnea While the incidence of true anaphylaxis to penicillins is relatively rare, sudden dyspnea could be related to narrowing of the airway from bronchoconstriction and angioedema, which require immediate action by the nurse. NOT: Maculopapular rash Maculopapular rash occurs in about 5%-10% of people who take ampicillin, but it is different from a true allergy. The nurse should follow up on the rash, but the symptom of dyspnea is aligned with anaphylaxis and is the priority. Sudden-onset headache The nurse should investigate a sudden-onset headache, but it is not likely related to the administration of ampicillin. Elevation in blood pressure An elevation in blood pressure is not associated with ampicillin. If the client has a significant drop in blood pressure, this could support possible anaphylaxis if other symptoms are also present.

Lisinopril

Angiotensin-converting enzyme (ACE) inhibitors treat congestive heart failure and hypertension. Many clients cannot tolerate a dry cough associated with ACE inhibitors and must be switched to an angiotensin II receptor blocker for the management of congestive heart failure. The healthcare provider prescribes 10 mg of lisinopril oral daily for a client with congestive heart failure. The nurse informs the client about which common side effect of lisinopril? Dry cough Dry cough and hypotension are the most common side effects associated with angiotensin-converting enzyme inhibitors, such as lisinopril. NOT: Weight gain A weight gain of 2-3 lb (1-1.4 kg) overnight or 5 lb (2.3 kg) in 1 week is a symptom of worsening heart failure; it is not associated with lisinopril. Dry eyes Dry eyes are not associated with lisinopril, but a loss of taste is associated with this medication. Confusion Neurological side effects of lisinopril include fatigue, dizziness, mood changes, and headaches. Confusion is not associated with lisinopril.

Propranolol (Inderal)

As a nonselective beta blocker, propranolol inhibits β2 receptors in the lungs, increasing the risk of bronchospasm and bronchoconstriction for the client with asthma, chronic obstructive pulmonary disease, or a history of bronchospasm. The nurse carefully educates the client to report new or worsening symptoms of bronchospasms such as wheezing, chest tightness, coughing, or fatigue. A client with hypertension and asthma takes multiple oral medications. Which medication does the nurse recognize as placing the client at risk for bronchospasm? Propranolol 80 mg Propranolol, a nonselective beta blocker, inhibits β2 receptors in the lungs, leading to bronchospasm and bronchoconstriction. Clients with asthma are at an increased risk for bronchospasm and constriction from beta blockers, aspirin, and nonsteroidal anti-inflammatory drugs. NOT: Lisinopril 5 mg Lisinopril is associated with a dry, hacking cough, but it is not known to cause bronchospasm. Amlodipine 5 mg Amlodipine is associated with hypotension, palpitations, and dyspnea, but it is not associated with bronchospasm. Spironolactone 25 mg Spironolactone may cause hyperkalemia, headache, dizziness, gastrointestinal upset, gynecomastia, and menstrual irregularities, but it is not associated with bronchospasm.

Atropine

Atropine is the initial drug of choice for symptomatic bradycardia caused by medications, inferior wall myocardial ischemia, or atrioventricular block (AV) with narrow QRS complexes. Bradycardia is defined as a heart rate of less than 60 beats/min. Hypotension, dizziness, and blurred vision are all symptoms of symptomatic bradycardia. While administering atropine, another nurse prepares one of three second-line treatments for symptomatic bradycardia, including external pacing, dopamine infusion, or epinephrine infusion. Note that for bradycardia caused by complete heart block, or second-degree AV block with widened QRS, the initial treatments of choice are pacing, dopamine infusion, and epinephrine infusion. A client with inferior myocardial ischemia has a heart rate of 34 beats/min, a blood pressure of 105/60 mm Hg, dizziness, and blurred vision. The nurse prepares to emergently administer which IV medication first? Atropine Atropine is the initial drug of choice for symptomatic bradycardia, such as occurs during an inferior wall myocardial infarction. NOT: Dopamine Dopamine infusion is one of the second treatments of choice for symptomatic bradycardia according to 2015 American Heart Association guidelines. It also maintains adequate blood pressure during shock. Epinephrine Epinephrine infusion is one of the second treatments of choice for symptomatic bradycardia according to 2015 American Heart Association guidelines. Epinephrine is also used for ventricular fibrillation, flutter, and unstable tachycardia. Dobutamine Dobutamine increases cardiac output in heart failure, but it is not indicated for symptomatic bradycardia.

Gemfibrozil (Lopid) -- Fibric acid derivative

Because the client reports not having any health problems, asking about a disease process with minimal or no symptoms is a good place to start assessment. Gemfibrozil is a fibric acid derivative and works by activating an enzyme that breaks down triglyceride molecules. Knowing the mechanism of action of this medication, hypertension can be eliminated as a potential problem. Because an irregular heartbeat causes a symptom, this health problem can be eliminated and, because the client denies health problems, previous instruction on blood glucose assessment is unlikely. The health problem remaining is that of an elevated cholesterol level and is the reason for this medication to be prescribed. A client who denies having any health problems reports taking gemfibrozil. Which question does the nurse ask to determine the reason for the medication? "Have you been told that your cholesterol is too high?" Gemfibrozil is a fibric acid derivative used to lower cholesterol and triglycerides. Fibrates increase lipoprotein breakdown and clearance and prevent the release of fatty acids from adipose tissue. This medication is indicated in the treatment of elevated triglyceride levels and will raise the high-density lipoprotein level. NOT: "Have you been instructed to check your blood sugar?" Gemfibrozil is a fibric acid derivative used to reduce lipid levels. It has no effect on blood glucose levels and is not indicated for use to control diabetes. "Have you ever experienced an irregular heartbeat?" Gemfibrozil is a fibric acid derivative used to reduce lipid levels. It has no effect on heart rate. "Have you been told that you have high blood pressure?" Gemfibrozil is a fibric acid derivative used to reduce lipid levels. It has no effect on blood pressure.

Betamethasone (Celestone)

Betamethasone is a corticosteroid prescribed to reduce inflammation and swelling of the bronchial tissues in the client with COPD. Because corticosteroids have been shown to cause some birth defects in the developing fetus, pregnancy should be avoided while taking this medication. Even though skin atrophy is an adverse effect of betamethasone, the medication should not be abruptly stopped but, rather, slowly tapered to prevent the development of adrenal insufficiency. Betamethasone causes elevated blood glucose and hypertension as adverse effects, and actions should be taken to monitor for these health problems. Betamethasone does not work quickly and is not used as a rescue medication for bronchoconstriction and wheezing. A female client is prescribed betamethasone as treatment for chronic obstructive pulmonary disease (COPD). Which of the following client statements indicate that teaching about this medication was effective? Select All That Apply "I will monitor myself for symptoms of high blood sugar." Betamethasone elevates blood glucose as an adverse effect. Blood glucose levels should be monitored while taking this medication. "I will talk to my healthcare provider before I become pregnant." Betamethasone is a corticosteroid that has been shown to cause some birth defects. It is necessary to discuss any plans for pregnancy with the healthcare provider. "I will continue to take my medication for high blood pressure." Betamethasone is a corticosteroid that has hypertension as an adverse effect. Blood pressure medication could be continued while taking betamethasone. NOT: "I will use this medication as needed for wheezing." Betamethasone is used in the treatment of COPD to reduce inflammation and swelling of the bronchial tissues. It is not used as a rescue medication for wheezing. "I will stop taking this medication if my skin starts to thin." Betamethasone is a corticosteroid that has a side effect of skin atrophy or thinning. Corticosteroids should not be stopped abruptly but, rather, tapered over several days to prevent the development of adrenal insufficiency.

Bethanechol (Urecholine)

Bethanechol chloride is a cholinergic medication that produces muscarinic effects, primarily on the gastrointestinal tract. Side effects are caused by acetylcholine activation (overexcitation and hyperactivity of the gastrointestinal tract) and include nausea, belching, and abdominal cramping. Dry eyes and constipation are not expected side effects. A client with urinary retention is prescribed bethanechol chloride. Which of the following side effects does the nurse expect to assess in this client? Select All That Apply Nausea Bethanechol chloride is a cholinergic medication that produces muscarinic effects, primarily on the gastrointestinal tract. Nausea is an expected side effect of this medication. Belching Bethanechol chloride is a cholinergic medication that produces muscarinic effects, primarily on the gastrointestinal tract. Belching is an expected side effect of this medication. Abdominal cramps Bethanechol chloride is a cholinergic medication that produces muscarinic effects, primarily on the gastrointestinal tract. Abdominal cramping is an expected side effect of this medication. NOT: Dry eyes Side effects are caused by acetylcholine activation and include lacrimation, not dry eyes. Constipation Diarrhea and fecal incontinence can occur with this medication. Constipation is not expected.

biphosphate (hydrogen phosphate)

Bisphosphonates are used to prevent further bone demineralization and bone loss in osteoporosis. A bisphosphonate should be taken on an empty stomach to maximize absorption. To prevent esophageal erosion or irritation, the medication should be taken with a full glass of water, and the client should be instructed to remain in an upright position for at least 30 minutes after taking it. Antacids interfere with the absorption of bisphosphonates and should be spaced 1-2 hours apart to avoid this interaction. An adequate intake of calcium and vitamin D is essential to prevent any further bone loss and demineralization. A client with osteoporosis is prescribed a bisphosphonate medication. Which of the following directions does the nurse include when teaching the client about this medication? Select All That Apply "Take it on an empty stomach." A bisphosphonate should be taken on an empty stomach to maximize absorption. "Take it with a full glass of water." A bisphosphonate should be taken with a full glass of water to minimize esophageal erosion or irritation. "Maintain adequate calcium and vitamin D intake." Bisphosphonates are used to prevent any further demineralization and bone loss. An adequate intake of calcium and vitamin D is essential for prevention. NOT: "Lie down for 30 minutes after taking the medication." Bisphosphonates are used to prevent further bone demineralization and bone loss in osteoporosis. The client should be instructed to remain in an upright position for at least 30 minutes after taking a bisphosphonate to prevent esophageal erosion or irritation. "Take it with antacids." Antacids interfere with the absorption of bisphosphonates and should be spaced 1-2 hours apart to avoid this interaction.

Ceftriaxone (Rocephin)

Ceftriaxone is a third-generation cephalosporin. Because such cephalosporins are structurally and pharmacologically similar to penicillin, they share the same side and adverse effects. Effects on the central nervous system include seizures, lethargy, and depression. Diarrhea is a common side effect of cephalosporins. Arrhythmias are not identified as being an adverse effect of cephalosporins. A client receives IV ceftriaxone for pneumonia. Which findings indicate to the nurse that the client is experiencing an adverse effect from this medication? Select All That Apply Depression Depression is considered a central nervous system effect of ceftriaxone. Lethargy Lethargy is considered a central nervous system effect of ceftriaxone. Seizures Seizures are considered a central nervous system effect of ceftriaxone. NOT: Arrhythmias Arrhythmias are not side or adverse effects of cephalosporins. Diarrhea Diarrhea is a common side effect of ceftriaxone and does not affect the central nervous system.

Cholestyramine (Questran)

Cholestyramine is a bile acid sequestrant, meaning the medication binds bile acids that contribute to total body cholesterol levels and eliminates them from the body through bowel movements. This medication is often used by those who are unable to tolerate statin medications for the treatment of hyperlipidemia. It can also be used in conjunction with a statin to enhance the reduction of total body cholesterol levels. Although clients with type 2 diabetes mellitus and hypertension may have elevated cholesterol levels as a part of or a contributing factor to the health problem, this medication has no direct effect on blood glucose levels or blood pressure control. Cholestyramine has no effect on heart rhythm. During an assessment, the nurse learns that a client who denies any health problems has been taking cholestyramine for 3 years. For which health problem does the nurse create a care plan? Hyperlipidemia Cholestyramine is one of three bile acid sequestrants used to block the absorption of bile acid in the small intestine and reduce the amount of cholesterol and low-density lipoproteins. NOT: Hypertension Cholestyramine is a bile acid sequestrant that blocks the absorption of bile acids in the small intestine to reduce total body lipid levels. Controlling lipid levels is an important step in reducing the progression of hypertension, but the fact that the client takes an antilipid medication does not indicate hypertension. Atrial fibrillation Cholestyramine is a bile acid sequestrant that limits the amount of bile acids absorbed from the small intestine and synthesized in the liver. This medication has no effect on heart rhythm. Type 2 diabetes Cholestyramine is a bile acid sequestrant that reduces the amount of bile acid in the body to reduce lipid levels. The combination of diabetes and hyperlipidemia would be concerning for the development of coronary artery disease, but the client's use of cholestyramine does not indicate diabetes.

Cisplatin (Platinol)

Cisplatin is a platinum-containing, alkylating antineoplastic agent that causes death of rapidly replicating cells. This effect produces the classic chemotherapy symptoms of nausea, vomiting, and anorexia due to the effects on the gastrointestinal epithelium and myelosuppression, resulting in pancytopenia. Side effects particular to this class of antineoplastics are nephrotoxicity, peripheral neuropathy, and ototoxicity. Leukocytosis and bradycardia are not associated with cisplatin. A client has been administered cisplatin. The nurse monitors for which of the following potential side effects? Select All That Apply Tinnitus Cisplatin is ototoxic and may produce changes in hearing, such as tinnitus. Increased creatinine Cisplatin is nephrotoxic, so the nurse monitors for increases to the client's baseline creatinine level. Numbness and tingling Cisplatin is nephrotoxic, and it can result in peripheral neuropathy, so the nurse should inquire about changes in sensation. NOT: Leukocytosis Cisplatin can increase the risk for infection, but this is due to myelosuppression, so the client's white blood cell count would be low, not elevated. Bradycardia Bradycardia is not associated with cisplatin.

Cyclosporine

Clients who are immunocompromised due to treatment with an agent such as cyclosporine or due to an underlying pathology must be observed closely for evidence of infection. Due to the compromised immune response, infection can quickly progress to sepsis and septic shock, making the elevated temperature the finding that requires the most immediate response by the nurse. Prior to notifying the healthcare provider, the nurse should assess other vital signs and for evidence of organ dysfunction. A nurse cares for a client prescribed cyclosporine. Which assessment finding does the nurse prioritize as requiring immediate intervention? Temperature of 101.3° F (38.5° C) Cyclosporine is an immunosuppressant. Any evidence of infection must be acted on immediately. If the client has evidence of sepsis, antibiotics should be administered within 30 minutes. NOT: Serum creatinine of 2 mg/dL (176.8 µmol/L) The client's elevated creatinine level (normal range is 0.5-1.2 mg/dL, depending on gender) should be reported, as cyclosporine can cause acute kidney injury. White blood cell count of 2,500/mm3 (2.5 × 109/L) Although neutropenia should be reported, this result indicates an increased risk for infection. The elevated temperature, which indicates a client's systemic response to an infection, is a more significant finding. Platelet count of 70,000/mm3 (70 × 109/L) This low platelet count may be related to myelosuppression by the medication and can increase the client's risk for bleeding, but this is only an indication of risk rather than an actual change in the client's condition. Cyclosporine is an immunosuppressant with a number of indications, including preventing organ transplant rejection and treatment of autoimmune conditions. The client should be advised of gingival hyperplasia as a possible side effect and to practice good oral hygiene and regular dental care. Acarbose, allopurinol, and atenolol are not associated with gingival hyperplasia. A client has developed gingival hyperplasia. Which medication prescribed to the client does the nurse disclose as a possible cause of this side effect? Cyclosporine Cyclosporine often results in gingival hyperplasia. The cause of this is poorly understood. NOT: Acarbose Acarbose, an oral antidiabetic agent, is not associated with gingival hyperplasia. Atenolol Atenolol, a cardioselective beta blocker, is not associated with gingival hyperplasia. Allopurinol Allopurinol, an antigout medication, is not associated with gingival hyperplasia.

Colchicine (Colcrys)

Colchicine is an antigout medication that reduces pain and swelling by inhibiting the migration of neutrophils into the area of inflammation. It alters the function of the ileal mucosa and interferes with intrinsic factor, causing malabsorption of vitamin B12. Colchicine does not decrease the intestinal absorption of vitamins B1, C, or K. A client with gout is prescribed colchicine. Which vitamin does the nurse instruct the client to take more of? Vitamin B12 Colchicine is an antigout medication that inhibits the migration of neutrophils into the area of inflammation to reduce pain and swelling. The nurse should instruct the client to take more vitamin B12, as colchicine causes malabsorption of that vitamin. NOT: Vitamin C Colchicine does not decrease the intestinal absorption of vitamin C. Vitamin K Colchicine does not decrease the intestinal absorption of vitamin K. Vitamin B1 Colchicine does not decrease the intestinal absorption of vitamin B1.

Dexamethasone (Decadron)

Dexamethasone is a corticosteroid that reduces inflammation in the brain. When a therapeutic response is achieved, the client's neurological status improves. The Glasgow Coma Scale (GCS) establishes baseline data on a client's neurological function. GCS scores range from 3 to 15, with categories for eye-opening response, verbal response, and motor response. This client shows improved neurological function by stating that pain is "blue" on a scale of 1-10. Although it is an inappropriate response, it is an intelligible word, so it indicates improved neurological function. The goal of dexamethasone is decreased cerebral edema accompanied by improved neurological function. A client with cerebral edema after a motor vehicle accident opens their eyes to noise, grunts when the nurse asks questions, and withdraws from a painful stimulus. After administering dexamethasone, the nurse monitors the client for what therapeutic response? The client states a pain level of "blue" on a scale of 1-10. The client responded to the nurse's question with an inappropriate but intelligible word. This shows neurological improvement, an expected therapeutic response from dexamethasone, because the client only grunted to the nurse's question before the dexamethasone. NOT: The client sleeps through the next peripheral blood draw. The client sleeping through painful stimuli (such as a peripheral blood draw) shows decreasing neurological function compared to before the dexamethasone administration. The nurse expects dexamethasone to improve neurological function. Both the client's feet and hands curl to the middle. A decorticate posture, or flexion of the arms and feet to the middle, indicates worsening motor neurological function. The nurse expects dexamethasone to improve neurological function. The nurse must open the client's eyelids to check the pupils. The client being unable to open their eyelids shows decreased neurological function. The nurse expects dexamethasone to improve neurological function.

Doxycycline (Vibramycin)

Doxycycline is a broad-spectrum tetracycline antibiotic that inhibits bacterial protein synthesis. To reduce the potential for gastrointestinal irritation, the medication should be taken with a full meal. The medication is prescribed to be taken once a day, which means the doses will be 24 hours apart. Foods and drugs that contain calcium can significantly decrease the absorption of doxycycline and should not be taken with it. Antacids can significantly decrease the absorption of doxycycline and should not be taken with this medication. A client taking doxycycline once a day for acne vulgaris experiences gastrointestinal irritation. Which recommendation does the nurse make to this client? "Take the medication with a full meal." Doxycycline is a broad-spectrum tetracycline antibiotic that inhibits bacterial protein synthesis. To reduce the potential for gastrointestinal irritation, this medication should be taken with a full meal. NOT: "Take the medication with an antacid." Antacids interact with doxycycline and significantly decrease the absorption of the medication. "Take the doses 12 hours apart." The medication is prescribed to be taken once a day. The doses will be 24 hours apart. "Take the medication with milk." Because foods and drugs containing calcium can significantly decrease the absorption of doxycycline, this medication should not be taken with milk. Medications that cause gastrointestinal upset, such as doxycycline, can lead to stomach upset, and this symptom is pronounced when the medication is taken on an empty stomach. Oral iron preparations are another common example of a drug ideally taken on an empty stomach for best absorption. Antacids interfere with the absorption of doxycycline and other medications. Taking doxycycline with water or at bedtime will not relieve gastrointestinal upset. A client prescribed doxycycline was told to take the medication on an empty stomach but finds it is causing stomach upset. How does the nurse respond? "You can take it with meals, but absorption may be somewhat reduced." Medications that cause gastrointestinal upset, such as doxycycline, should ideally be taken on an empty stomach for absorption but, if the client cannot tolerate the side effect, the client is advised to take the medication with food. NOT: "Because the medication is prescribed on an empty stomach, take it with a full glass of water." Taking the medication with water will not reduce the symptom of stomach upset. "Take the medication at bedtime when the nausea is less likely to be pronounced." The client will still experience stomach upset taking doxycycline at bedtime, and it may interfere with sleep, as well. "Take an over-the-counter antacid prior to the medication to reduce stomach upset." Antacids should be avoided when taking other medications, as they interfere with absorption.

Heparin

Heparin is a natural anticoagulant commonly used to treat a pulmonary embolism caused by deep vein thrombosis. It is administered through a continuous IV infusion. The major adverse effect of heparin is bleeding, which can occur anywhere within the cardiovascular, pulmonary, genitourinary, or gastrointestinal tract areas. Tarry, or black, sticky stools, are an indication of bleeding in the gastrointestinal tract and should be immediately reported to the healthcare provider. A client receives IV heparin as treatment for a pulmonary embolism. For which potential complication does the nurse monitor this client? Tarry stools Heparin is an anticoagulant that inhibits the conversion of prothrombin to thrombin. This effect increases the risk for bleeding. Bleeding in the gastrointestinal tract causes tarry stools and is considered a major complication of heparin therapy. NOT: Dyspnea Dyspnea, or shortness of breath, is a major symptom of a pulmonary embolism. This is not a complication of heparin therapy. Lethargy Lethargy can occur from a pulmonary embolism because of insufficient oxygenation to cerebral tissues. This is not a complication of heparin therapy. Hyperglycemia Hyperglycemia is an elevation in blood glucose level. This is not a complication of heparin therapy. Heparin is monitored by the activated partial thromboplastin time (aPTT), which measures the efficacy of the intrinsic and common coagulation pathways to determine the effect of heparin on clotting. Platelets are needed for blood clotting, but they are not used to monitor the effectiveness of heparin therapy. International normalized ratio and prothrombin time (PT) are used to evaluate the effectiveness of warfarin. A client with a pulmonary embolism is treated with continuous IV heparin. Which laboratory test does the nurse use to evaluate the effectiveness of this client's treatment? Activated partial thromboplastin time Activated partial thromboplastin time is a laboratory value that measures the efficacy of the intrinsic and common coagulation pathways to determine the effect of heparin on clotting. NOT: International normalized ratio International normalized ratio is a laboratory value used to evaluate the effectiveness of warfarin. Platelet count Platelets are required for effective blood clotting, but monitoring this blood product does not evaluate the effectiveness of heparin therapy. Prothrombin time Prothrombin time is a laboratory value used to evaluate the effectiveness of warfarin. IV heparin is used in the treatment of pulmonary embolism with the dose adjusted according to the aPTT. The therapeutic range for treatment of this health problem is 1.5-2.5 times the control value. Because the client's value is 60 seconds, the current dose is considered therapeutic and should be continued. Because the aPTT is at a therapeutic level, there is no reason to stop the infusion for 1 hour, contact the healthcare provider, or provide protamine sulfate (the antidote to heparin). A client receiving IV heparin for a pulmonary embolism has an activated partial thromboplastin time (aPTT) of 60 seconds. Which action does the nurse take? Continues the infusion as prescribed The effectiveness of heparin is monitored through the aPTT, which should be 1.5-2.5 times the normal value. Because the standard control is 30 seconds and the client's value is 60 seconds, the current dose is considered therapeutic and should be continued. NOT: Stops the infusion for 1 hour There is no reason to stop the infusion for 1 hour. The client's aPTT is 60 seconds, which is considered therapeutic. Administers protamine sulfate Protamine sulfate is the antidote to heparin. It is not required because the aPTT is at a therapeutic level. Notifies the healthcare provider There is no reason to notify the healthcare provider because the aPTT is at a therapeutic level.

Lovastatin (Mevacor)

Lovastatin, a β‑hydroxy β‑methylglutaryl-coenzyme A reductase inhibitor (or statin), is a category X medication and contraindicated with pregnancy and breastfeeding. Category X medications have been shown to cause serious abnormalities of the fetus in both humans and animals, so a client planning to become pregnant should discontinue lovastatin. Modifiable risk factors that reduce the risk of atherosclerosis and cardiac disease include exercising, smoking cessation, and eating a diet high in fruits, vegetables, and whole grains. A client takes lovastatin for hyperlipidemia. The nurse tells the client to inform the healthcare provider if planning what lifestyle change? Becoming pregnant All statins, including lovastatin, are pregnancy category X. A client who is planning to become pregnant, is pregnant, or is breastfeeding should not take statin medications. NOT: Beginning exercise Moderate exercise 3-4 times per week for about 40 minutes may help reduce levels of low-density lipoprotein cholesterol for a client with hyperlipidemia, in conjunction with lovastatin. Becoming a vegetarian A diet high in vegetables, legumes, and whole grains may help with reducing levels of low-density lipoprotein cholesterol for a client with hyperlipidemia, in conjunction with lovastatin. Quitting smoking Quitting smoking decreases a client's risk of atherosclerosis and cardiac disease. The nurse should ask the client if any assistance is needed with quitting smoking.

Magnesium hydroxide (Milk of Magnesia)

Magnesium hydroxide is considered a saline cathartic antacid that causes osmotic retention of fluid. This action distends the colon and causes peristaltic activity that can lead to diarrhea. Magnesium hydroxide can cause hypotension but not hypertension. It does not cause muscle cramping and has no effect on the urinary bladder that might cause urinary retention. A client reports taking magnesium hydroxide for gastric reflux. For which side effect of this medication does the nurse assess this client? Diarrhea A common side effect of magnesium hydroxide is diarrhea. This medication causes osmotic retention of fluid to distend the colon, leading to peristaltic activity. NOT: Hypertension Hypertension is not identified as a common side effect of magnesium hydroxide, although it can cause hypotension. Urinary retention Magnesium hydroxide is not identified as affecting urine output or having any effects on the bladder that might cause urinary retention. Muscle cramps Magnesium hydroxide is not identified as causing muscle cramps.

Gentamicin (Garamycin)

Many ophthalmic solutions cause eye irritation. As with all medications, it is important that the nurse educate the client about what are considered normal effects to reduce client concerns and to prevent unnecessary inquiries to the healthcare provider. Not allowing someone else to use the applicator or to let it touch the eye are correct, as is washing hands before administering the medication. A client is prescribed gentamicin ophthalmic drops. Which statement by the client requires the nurse to provide additional information? "I should contact my healthcare provider if the drops cause burning or itching." Irritation from the eye drops is an expected effect of the the medication, so there is no need to report it to the healthcare provider. OK: "I should avoid touching the tip of the applicator to my eye." The client should not touch the applicator to any part of the eye to avoid contamination of the solution. "I should not allow anyone else to use the eye drop applicator." To avoid contamination of the solution, the medication applicator should only be used by the client. "I should wash my hands before and after administration." Handwashing before and after instillation reduces the risk of introducing pathogens.

Mercaptopurine (Purinethol)

Mercaptopurine is an oral purine antagon that interferes with DNA synthesis. This makes it an effective antimetabolite and antineoplastic that also affects rapidly dividing healthy cells, such as bone marrow and the gastrointestinal epithelium. Antimetabolites' effects on bone marrow include decreased levels of red blood cells, leading to anemia and fatigue; decreased levels of white blood cells (leukopenia), leading to an increased risk for infection; and decreased platelet levels (thrombocytopenia), leading to an increased risk for bleeding. When all three blood cell levels are low, it is referred to as pancytopenia. The effects of antimetabolites on gastrointestinal epithelial cells lead to anorexia, nausea, or vomiting. Fluids should not be limited. Elevating the legs is not an action associated with taking antimetabolites. A client has been treated with mercaptopurine. Which of the following actions does the nurse incorporate into the plan of care? Select All That Apply Monitor intake and output. Side effects include an alteration of gastrointestinal mucosa that can lead to anorexia, nausea, or vomiting. Vomiting can lead to dehydration, so the nurse should monitor hydration status. Cluster care and rest periods. Due to myelosuppression, anemia will result in fatigue. To help the client cope, the nurse should cluster care and rest periods. Avoid invasive procedures. Due to the high risk for thrombocytopenia and leukopenia, the nurse should avoid invasive procedures, such as injections and suppositories. NOT: Elevate the legs several times a day. Elevating the legs several times a day promotes the absorption of peripheral edema and minimizes nocturnal issues, such as paroxysmal dyspnea and nocturia. This intervention is not related to taking an antimetabolite. Limit fluid intake after meals. The client's fluid intake should not be limited.

Metronidazole (Flagyl)

Metronidazole is an antitrichomonal amebicide that is used to treat clients with C. diff. It is contraindicated for use in clients with active central nervous system disease and should be used with caution in those with a seizure disorder. Metronidazole is not contraindicated for use in a client with a hallucinogenic disorder. Constipation is an adverse effect of metronidazole. The medication should be used with caution in those with heart failure. It has an adverse cardiovascular effect of flattening the T wave during an electrocardiogram. A client with Clostridium difficile (C. diff) is prescribed IV metronidazole. For which concurrent health problem does the nurse question the use of this medication? Seizure disorder Metronidazole should be used with caution in a client with a seizure disorder. This medication lowers the seizure threshold, resulting in increased seizure occurrences. NOT: Constipation Constipation is a side effect of metronidazole. It is not a concurrently occuring health problem but, rather, a result of the medication. Torsades de pointes Metronidazole is not identified as being contraindicated for use in a client with torsades de pointes, although the medication should be used cautiously in those with heart failure. It can cause flattening of the T wave. Hallucinations Metronidazole is not identified as being contraindicated for use in a client experiencing hallucinations.

Niacin (B3)

Niacin, a water-soluble vitamin, decreases lipoprotein and triglyceride synthesis. Common side effects include flushing of the face and neck, rash, pruritus, blurred vision, abdominal discomfort, hyperglycemia, and liver toxicity. Niacin doses may start small to minimize adverse effects and then be increased according to client tolerance. The healthcare provider prescribes niacin for a client with hyperlipidemia. The nurse determines that the client understands niacin education by stating which of the following? Select All That Apply "I will notify my healthcare provider if I experience blurry vision." Blurred vision occurs as an adverse effect of niacin. The client should immediately notify the healthcare provider of new-onset blurred vision. "I will notify my healthcare provider if I experience palpitations." Palpitations are not typically associated with niacin, but they may occur with an allergic reaction to niacin. Best practice includes immediately instructing the client to report new-onset palpitations to the healthcare provider for further assessment of the condition. "I will take aspirin 30 minutes before my niacin." Niacin causes flushing. Educate the client to try taking a nonsteroidal anti-inflammatory drug or aspirin 30 minutes before niacin to reduce cutaneous flushing. "I take niacin immediately after eating a meal." Because niacin may cause abdominal discomfort, instruct the client to take it after meals. NOT: "I take lozenges to help my niacin-related cough." A dry, nagging cough may occur with an angiotensin-converting enzyme inhibitor, such as lisinopril, but it is not associated with niacin.

Ophthalmic fluorescein

Ophthalmic fluorescein strips are used to detect corneal defects or abrasions by introducing a luminescent dye to the cornea. An impregnated tip of the strip is moistened and then applied to the conjunctiva or fornix. After the client blinks several times, distributing the dye across the cornea, an ultraviolet light is used to visualize the corneal defects that have been stained by the fluorescein dye. The other directions to the client are not helpful for corneal testing with fluorescein strips. A client is being screened for corneal abrasions using a fluorescein strip. The nurse offers which direction to the client for this procedure? "Blink several times after you feel the strip touch your eye." For best results, the client should blink several times after application. This distributes the dye over the surface of the cornea. NOT; "Look up and away, and hold your gaze as long as possible." The client needs to blink. Because the strip will touch the eye, blinking will be unavoidable. "Hold the strip in place for 5 minutes, and then remove it for assessment." The strip is only touched to the cornea by the nurse and removed. The dye present on the strip dissipates onto the surface of the eye. "Close your eyes and hold them shut until directed to open them." Holding the eyes closed will not allow for distribution of the dye.

Phenelzine (Nardil)

Phenelzine, in the monoamine oxidase inhibitor (MAOI) class of drugs, is an antidepressant used for resistant forms of depression. Due to potential serious adverse effects, these medications must be monitored closely. One adverse effect is hypertensive crisis, which can occur if the MAOI is taken with stimulant medications or with substances containing tyramine (found in many common foods and beverages). Another serious side effect occurs if the client takes an MAOI with another drug that elevates serotonin levels, such as selective serotonin reuptake inhibitors. This syndrome results in fever, agitation, hyperreflexia, tremor, diaphoresis, dilated pupils, and diarrhea. A client is newly prescribed phenelzine. Which of the following interventions does the nurse include in the client's plan of care? Select All That Apply Increase frequency of blood glucose monitoring. Monoamine oxidase inhibitors, such as phenelzine, have been shown to increase insulin sensitivity, putting a client at risk for hypoglycemia. Request an order for a tyramine-free diet. The client must avoid tyramine-containing foods and beverages. Generally, this includes high-protein foods that have undergone protein breakdown by aging, fermenting, pickling, or smoking. Sausage, salami, beer, wine, and sauerkraut are some examples. In addition, excessive amounts of caffeine and chocolate should be avoided. Avoid administering opioid analgesics. Phenelzine is a monoamine oxidase inhibitor that increases serotonin levels. Opioids enhance the serotonergic effect of these inhibitors and could result in serotonin syndrome. NOT: Ensure fluid intake of 1,500 mL daily. A volume of 1,500 mL is a fluid restriction compared to the normal recommended intake of about 3 L and is not indicated for clients taking monoamine oxidase inhibitors. Clients should avoid dehydration due to the risk for orthostatic hypotension and to ensure adequate renal clearance. Avoid administering aspirin-containing medication. Although monoamine oxidase inhibitors have numerous serious drug-drug interactions, aspirin is not contraindicated. Both stimulants and medications that affect serotonin levels should be avoided due to the serious complication of serotonin syndrome.

Phenylephrine (Neo-Synephrine)

Phenylephrine, an α agonist, stimulates α receptors resulting in vasoconstriction and increased blood pressure. Clients with shock or hypotension may benefit from phenylephrine. The nurse frequently monitors the client's vital signs, including heart rate and blood pressure, during the administration of any adrenergic drugs and adjusts the drip rate according to the HCP's prescription to treat hypotension and prevent hypertension. The healthcare provider (HCP) prescribes a phenylephrine IV infusion for a client with hypotension. The nurse initiating the client's phenylephrine infusion should take which action? Frequently monitor the client's blood pressure. The client's blood pressure must be frequently monitored to titrate the medication to the desired effect. Often, the HCP will establish goals or parameters to guide titrations. NOT: Notify the HCP when the infusion is completed. A prescription for a continuous infusion, such as that for the adrenergic drug phenylephrine, requires the nurse to hang a new bag of phenylephrine when the infusion is completed. The nurse only stops the infusion if the HCP discontinues it or the client meets the prescribed parameters to discontinue the infusion. Protect the solution from light while infusing it. Phenylephrine requires protection from light while in storage up until the time of use. While being infused, phenylephrine does not require protection from light. Obtain a baseline blood glucose level. Phenylephrine administration does not require a baseline blood glucose level.

Phenytoin (Dilantin)

Phenytoin, a hydantoin anticonvulsant, works by modulating neuronal sodium and calcium channels. The therapeutic phenytoin level is 10-20 μg/mL. Because phenytoin is bound to proteins, clients with low protein levels (such as those with chronic renal failure or malnourishment) may be prone to toxic phenytoin levels. Common side effects at therapeutic dilantin levels include lethargy, confusion, abnormal movements, diarrhea, hypotension, acne, gingival hyperplasia, rash, and hirsutism. At toxic levels, the client may experience symptoms such as ataxia, nystagmus, slurred speech, or difficulty speaking (dysarthria). A client with chronic renal failure and a history of seizures has a phenytoin level of 28 μg/mL. The nurse monitors the client for which symptom? Select All That Apply Ataxia The nurse monitors for ataxia, or loss of voluntary muscle coordination and control, in a client with toxic phenytoin levels. Nystagmus The nurse monitors for nystagmus (involuntary back-and-forth eye movements) in this client with toxic phenytoin levels. NOT: Agitation Lethargy and confusion are common side effects with normal phenytoin levels. Agitation is not typically associated with phenytoin. Constipation Antiepileptic drugs, such as phenytoin, are associated with diarrhea, not constipation. Hypertension Phenytoin may cause hypotension, not hypertension, as an adverse effect.

Piperacillin/Tazobactam (Zosyn)

Piperacillin/tazobactam is an extended-spectrum drug. It has a wider range of activity than other penicillins by preventing the breakdown of the penicillin by the β‑lactamase enzyme. Because of this, piperacillin/tazobactam is indicated in the treatment of pneumonia. Penicillin medications are usually well tolerated unless an allergy exists. The most common reaction is urticaria, followed by the unpredictable reaction of a maculopapular rash. Confusion, bradycardia, and constipation are not identified as being adverse effects of piperacillin/tazobactam. A client with no known drug allergies receives IV piperacillin/tazobactam as treatment for pneumonia. For which of the following potential serious side effects does the nurse assess this client? Select All That Apply Urticaria Urticaria is a common symptom of a type 1 hypersensitivity reaction to piperacillin/tazobactam. Rash A maculopapular rash is considered an unpredictable drug reaction to piperacillin/tazobactam. NOT Confusion Confusion is not identified as a side effect of piperacillin/tazobactam. Bradycardia Bradycardia is not identified as a side effect of piperacillin/tazobactam. Constipation Constipation is not identified as a side effect of piperacillin/tazobactam. Though anaphylactic reactions to medications are rare, penicillin allergies are one of the more common drug allergies. The combination of hives, pruritus, and difficulty breathing support this conclusion. Epinephrine administered IM is the treatment of choice in anaphylaxis. It counteracts the vasodilation and increased capillary permeability caused by histamine (increasing blood pressure and making vessels less leaky). It also results in bronchodilation, which helps to open airways. When given IM, the onset is rapid (5 minutes) and of short duration, which allows it to be repeated in 15 minutes if necessary. Additional interventions might include oxygen delivery, steroid administration, and nebulized racemic epinephrine. A client is receiving an IV infusion of piperacillin-tazobactam. The nurse notes the development of hives, and the client states, "I am having trouble breathing and am itchy all over." What priority action does the nurse take? Administers epinephrine as prescribed Administering epinephrine is the priority action for a client with evidence of anaphylaxis. NOT: Administers diphenhydramine as prescribed Diphenhydramine can be given for the anaphylaxis, but the priority is epinephrine, which has a more immediate effect on the histamine response compared to diphenhydramine. Checks blood pressure and oxygen saturation Hypotension occurs in anaphylaxis because of the massive vasodilation caused by the response to histamine, but even if the client is not yet hypotensive, the nurse will still administer epinephrine to offset the effects of histamine. Assesses the client's airway and breathing The client has just spoken to the nurse, so the nurse knows the client has a patent airway.

Prazosin (Minipress)

Prazosin, an α1 adrenergic antagonist that treats benign prostatic hypertrophy as well as hypertension, often causes severe hypotension after the first dose. Dizziness and orthostatic hypotension are common side effects, even after the first dose. The client starts at a low dose and titrates the dose up as tolerated. The healthcare provider prescribes prazosin for a client with newly diagnosed benign prostatic hypertrophy. The nurse instructs the client that the side effects of prazosin can be avoided by taking the medication in which way? Before bedtime The nurse advises the client to take prazosin before bedtime to avoid the common side effects of syncope, dizziness, and lightheadedness. NOT: Before exercising The client should avoid taking prazosin before exercise, as prazosin may cause hypotension. With a diuretic Taking prazosin with a diuretic is not recommended. The client should take a diuretic in the morning to avoid nocturia and prazosin at night to avoid orthostatic hypotension and daytime fatigue. With a full meal Taking prazosin with a full meal does not decrease the primary side effect, hypotension.

Procainamide (Pronestyl)

Procainamide is a Class Ia antiarrhythmic medication used to regulate atrial fibrillation, premature atrial and ventricular contractions, and ventricular tachycardia. This medication affects the vagal response and can cause heart rate and blood pressure to drop. Prior to giving this medication, it is essential to have a baseline blood pressure and heart rate to evaluate the effectiveness of the medication. BUN and creatinine, hemoglobin and hematocrit, and urine output and osmolality are all levels that should be assessed in an acutely ill client, but these levels have no direct impact on the administration of procainamide. Urine output may be decreased due to low blood perfusion of the kidneys caused by decreased cardiac output. Low hemoglobin may contribute to certain arrhythmias. Elevated BUN and creatinine may require an adjusted dose of procainamide. A client is prescribed procainamide. What does the nurse assess before giving this medication? Blood pressure and heart rate Drugs that are categorized as Class Ia have anticholinergic activity and can cause or worsen hypotension and bradycardia. Prior to giving this medication, heart rate and blood pressure should be assessed to serve as a baseline when monitoring the response to the medication. NOT: Hemoglobin and hematocrit Hemoglobin and hematocrit are levels used to assess perfusion and oxygenation, but they have no direct effect on the administration of procainamide. BUN and creatinine BUN and creatinine levels are used to evaluate kidney function. Although this is an important parameter to assess, these levels have no direct effect on the administration of procainamide. Urine output and osmolality Urine output and osmolality are used to assess kidney function and certain electrolyte levels. Although these are important parameters to assess, they have no direct effect on the administration of procainamide.

Quinidine

Quinidine is a Class Ia antiarrhythmic medication. This type of medication reduces myocardial excitability in the ventricles and eliminates or reduces ectopic foci in the ventricles. It can affect vagal stimulation. Common side effects of Class Ia medications include tinnitus, diarrhea, hearing loss, and blurred vision. A significant side effect of this medication is the development of premature ventricular contractions, which is one of the most potentially life-threatening ventricular arrhythmias. A client with an arrhythmia caused by mitral valve prolapse receives IV quinidine. For which side effects does the nurse assess this client? Select All That Apply Blurred vision Quinidine can cause cinchonism, or effects of quinidine toxicity. Mild effects of toxicity include blurred vision. Hearing loss Quinidine can cause cinchonism, or effects of quinidine toxicity. Mild effects of toxicity include hearing loss. Diarrhea Quinidine can cause cinchonism, or effects of quinidine toxicity. Mild effects of toxicity include diarrhea. Tinnitus Quinidine can cause cinchonism, or effects of quinidine toxicity. Mild effects of toxicity include tinnitus. NOT: Ventricular tachycardia Quinidine affects the electrical activity of the heart and has indirect cholinergic effects. A significant adverse effect of this medication is premature ventricular contractions, which is considered one of the most potentially life-threatening ventricular arrhythmias.

Quinolones - moxifloxacin

Quinolones such as moxifloxacin, levofloxacin, and ciprofloxacin may cause a prolonged QT interval that places the client at risk for ventricular fibrillation and torsades des pointes ("twisting of the points"). A prolonged QT interval means the heart muscles are taking longer than normal to repolarize between beats and may not be ready to receive the next electrical impulse. Torsades de pointes is a polymorphic ventricular tachycardia that appears to be twisting around the electrocardiogram line. Many other medications can cause prolonged QT intervals, including certain antipsychotics, anticonvulsants, antihistamines, antiemetics, and antifungals. A nurse cares for a client taking moxifloxacin for the treatment of endocarditis. The nurse monitors the client's heart rhythm for which adverse effect of quinolone therapy? QT prolongation An adverse effect of quinolone therapy is prolonged QT, which places the client at greater risk for dysrhythmia, especially if the client already takes antidysrhythmic medications. NOT: Absent PR interval An absent PR interval may occur with atrial fibrillation or a junctional rhythm. Quinolone therapy does not increase the risk of atrial fibrillation or junctional rhythms. QRS widening QRS widening may occur with a bundle branch block or intraventricular conduction delay. Quinolone therapy does not increase the risk of a widened QRS. ST elevation ST elevation or depression may occur when the heart does not receive enough oxygen. Quinolone therapy is not associated with ST elevation or depression.

Vancomycin (Vancocin) -- Red Man Syndrome (RMS)

RMS is seen when vancomycin infusions are administered too rapidly. While bothersome, it is usually harmless and responds well to antihistamine therapy. It is characterized by flushing of the face, head, neck, and upper trunk areas, as well as by pruritus and possible hypotension. Vancomycin should be reconstituted as recommended with either 5% dextrose in water or normal saline and infused for at least 60 minutes. The nurse should assess the client's baseline skin tone and blood pressure prior to administration and ask the client to notify the nurse if they experience a warm flushing sensation or dizziness during the infusion. A nurse initiates an IV infusion of vancomycin. For which of the following signs or symptoms associated with red man syndrome (RMS) does the nurse monitor? Select All That Apply Pruritus RMS, thought to result from the release of histamine, is not a true allergic reaction. It can cause pruritus. Rash on the neck Redness of the face, neck, and back are classic signs of RMS and is thought to be related to the cutaneous reaction to histamine response. Flushing Flushing and redness are classic signs of RMS and are thought to be related to vasodilation from histamine response. NOT: Bradycardia Bradycardia is not associated with RMS. Hypertension In more severe cases, RMS can result in hypotension but not hypertension. Red man syndrome is seen when vancomycin infusions are administered too rapidly. While bothersome, this syndrome is usually harmless and responds well to antihistamine therapy. It is characterized by flushing of the face, head, neck, and upper trunk areas, as well as by pruritus and possible hypotension. The syndrome typically occurs with the first administration of the drug and when the infusion is too rapid. The drug should be stopped and antihistamines administered. The infusion may then be restarted at a slower rate. Vancomycin should be reconstituted as recommended with either 5% dextrose in water or normal saline and infused over at least 60 minutes. The nurse should assess the client's baseline skin tone and blood pressure prior to administration and ask the client to notify the nurse if they experience a warm flushing sensation or dizziness during the infusion. The nurse administers vancomycin IV. The client's face becomes flushed and red in appearance. Which of the following actions should the nurse take? Select All That Apply Administer prescribed diphenhydramine. The redness, while not a true drug allergy, is related to hypersensitivity. Diphenhydramine has been found to greatly relieve such symptoms. It is sometimes prophylactically administered to the client when receiving vancomycin if red man syndrome has been previously experienced. Assess the client's blood pressure. Due to vasodilation, a drop in blood pressure can be serious with red man syndrome, so it should be assessed after pausing the infusion. NOT: Assess the client's oxygen saturation. Red man syndrome does not cause a drop in oxygen saturation, so this is not a priority. Slow the rate of the vancomycin infusion. Flushing of the face and neck may be evidence of red man syndrome, which is related to rapid infusion rates of vancomycin. The priority when these signs are evident is stopping the infusion while additional interventions are taken to correct the reaction. The infusion may then be resumed at a slower rate. Remove the IV catheter. This syndrome is usually harmless and is directly related to the rate of infusion. If mild, the infusion may be able to be restarted, so the nurse should not remove the catheter.

Rifampin (Rifadin)

Rifampin is a macrocyclic antibiotic used in the treatment of Mycobacterium, the organism that causes tuberculosis. It is used either alone or with other antituberculosis medications. The most notable side effect of this medication is change in the color of urine, tears, sweat, sputum, and feces to a red, orange, or brown color. The nurse might recommend the client wear glasses to avoid the unpleasant discoloration of contact lenses. A client taking rifampin, isoniazid, and pyrazinamide for tuberculosis reports contact lenses being stained orange. Which action should the nurse perform to help this client? Explain that this is a usual side effect. Rifampin may cause bodily fluids to appear orange or red. This color change affects urine, tears, sweat, sputum, and feces. This color change is considered benign and does not need to be treated. NOT: Notify the healthcare provider. It is unnecessary to notify the healthcare provider because orange staining of contact lenses is an expected side effect of this medication. Administer a dose of vitamin K. Vitamin K is an antidote for warfarin, an anticoagulant, and is used when signs of bleeding are present. Orange staining of contact lenses does not indicate bleeding. Instruct the client to stop the medication. There is no reason for the client to stop the medication. Medications to treat tuberculosis should never be discontinued without consulting the healthcare provider. Orange staining of contact lenses when taking rifampin is an expected side effect. Rifampin is a synthetic macrocyclic antibiotic that kills Mycobacterium tuberculosis. It is considered a bactericidal in that it kills the organism that causes tuberculosis by inhibiting protein synthesis in the cell. The main mechanism of action is the inhibition of RNA and some DNA syntheses. Rifampin interacts with a variety of medications, including beta blockers, benzodiazepines, cyclosporine, oral anticoagulants, oral antihyperglycemics, phenytoin, quinidine, sirolimus, and theophylline. A client with tuberculosis is prescribed rifampin. What information does the nurse provide to the client about this medication? "It is a bactericidal for the tuberculosis organism." Rifampin is a bactericidal drug that kills the bacteria that cause tuberculosis by inhibiting protein synthesis. NOT: "It causes a blue tint to urine and saliva." Rifampin is a synthetic macrocyclic antibiotic that kills Mycobacterium tuberculosis. It can turn urine, sweat, tears, and saliva to orange, red, or brown. "It has no known interactions with other medications." Rifampin interacts with beta blockers, benzodiazepines, cyclosporine, oral anticoagulants, oral antihyperglycemics, phenytoin, quinidine, sirolimus, and theophylline. "It is not well understood how this medication works." Rifampin inhibits RNA synthesis and may affect DNA synthesis in the cell.

Simvastatin (Zocor)

Simvastatin, a β‑hydroxy β‑methylglutaryl-coenzyme A reductase inhibitor, also known as a statin, starts reducing levels of low-density lipoprotein cholesterol in the blood within 6-8 weeks of taking it. It is contraindicated with liver dysfunction. Common side effects of statins include headache, abdominal pain, and rash. The nurse monitors the client for muscle pain that can progress to rhabdomyolysis. Left untreated, rhabdomyolysis may cause renal failure and death. A client with coronary artery disease and hyperlipidemia takes simvastatin to manage cholesterol. The nurse questions the client regarding the presence of which expected effects? The nurse asks the client about the presence of which of the following common side effects? Select All That Apply Rash The most common side effects of statins include abdominal pain, headache, and rash. Headache The most common side effects of statins include abdominal pain, headache, and rash. Gastric upset The most common side effects of statins include abdominal pain, headache, and rash. Taking statin medications with 6-8 oz (177-236 mL) of water may help decrease abdominal pain. Muscle pain Muscle pain (an adverse effect of statin medication) increases the client's risk for rhabdomyolysis, a condition in which muscle protein is broken down and eliminated in the urine. Instruct the client to notify the healthcare provider immediately of muscle soreness or change in urine color. NOT: Peripheral edema Edema is not a common side effect of any antilipemic medications, including statins.

sodium polystyrene sulfonate (Kayexalate)

Sodium polystyrene sulfonate is used to remove potassium from the circulation by exchanging sodium ions for potassium ions in the large intestine. The solution needs to be retained for a minimum of 30-60 minutes, but several hours is preferable if possible. The expected effect is diarrhea that occurs once the medication is expelled. Sodium polystyrene sulfonate is not effective if expelled immediately after administration via enema. It takes longer than 5 minutes for the medication to be effective. A client with an elevated potassium level asks how long the sodium polystyrene sulfonate enema will need to be retained. Which response does the nurse provide to the client? "At least 30-60 minutes." Sodium polystyrene sulfonate is used to remove potassium from the circulation by exchanging sodium ions for potassium ions in the large intestine. Sodium polystyrene sulfonate needs to be retained for a minimum of 30-60 minutes, but several hours is preferable if possible. NOT: "Until the solution is administered." Sodium polystyrene sulfonate is not effective if expelled immediately after administration via enema. "Five minutes or more." Sodium polystyrene sulfonate is not effective if the solution is retained for only 5 minutes. "Until diarrhea starts." Sodium polystyrene sulfonate is not effective if the solution is retained for only a few minutes.

Colestipol (Colestid) -- bile acid sequestrant (BAS)

The body uses bile acids to reabsorb cholesterol. Colestipol, a bile acid sequestrant (BAS), binds to bile in the gastrointestinal tract and causes it to be excreted in the feces, effectively reducing low-density lipoprotein levels. BASs commonly cause constipation and may be used to treat certain types of diarrhea. A client with active liver disease may safely take colestipol, but the nurse should advise the client not to take other medications for 1 hour before and for 6 hours after taking a BAS. Bile acid sequestrant (BAS) is usually very chunky and sticky, it sticks to the GI wall and causes constipation. Bile (made of cholesterol) attaches to a bile acid sequestrant and get secreted in GI tract. bile cannot be reabsorbed and reused, the liver has to use cholesterol in the bloodstream to make more bile. In this way, the LDL in bloodstream decreased. The healthcare provider prescribes colestipol for a client with liver disease and hyperlipidemia. The nurse includes which common side effect of colestipol in the client's medication instructions? Constipation Bile acid sequestrants commonly cause constipation when used to lower cholesterol and may be used to treat certain types of diarrhea. NOT: Muscle pain Muscle pain may be an adverse reaction to a statin drug, but it is not a side effect of bile acid sequestrants. Rash Rash is a side effect of statin medications, but it is not a side effect of bile acid sequestrants. Edema No antilipidemic drugs in any class cause edema as a side effect.

Digoxin (Lanoxin)

The nurse carefully educates the client newly taking digoxin on the signs and symptoms of toxicity. The most common symptoms of digoxin toxicity include cardiac arrhythmias, bradycardia, nausea, vomiting, visual disturbances, headache, confusion, and dizziness. The nurse instructs the client to report symptoms of toxicity to the healthcare provider immediately. A healthcare provider prescribes a client with atrial fibrillation 0.25 mg of digoxin oral daily. The nurse educates the client to monitor for which of the following symptoms of digoxin toxicity? Select All That Apply Visual disturbances Visual disturbances associated with digoxin toxicity include seeing yellow-green halos and blurred vision. Headache Neurological symptoms of digoxin toxicity include confusion, headache, and dizziness. Nausea Gastrointestinal symptoms of digoxin toxicity include nausea and vomiting. Bradycardia Cardiovascular symptoms of digoxin toxicity include bradycardia and arrhythmias. NOT: Urinary retention Digoxin toxicity is not associated with urinary retention. Digoxin has a narrow therapeutic range and may become toxic if kidney function or electrolyte levels decrease. The nurse monitors the client's levels of digoxin, creatinine, magnesium, and potassium to assess for possible toxicity. The nurse cares for a client with heart failure who takes furosemide and digoxin. Which laboratory result indicates to the nurse that the client is at increased risk for digitalis toxicity? Potassium of 2.6 mEq/L Normal potassium levels range from 3.5 to 5 mEq/L. Low levels of potassium, or hypokalemia, put the client on digoxin at risk for digitalis toxicity. Furosemide causes potassium to be excreted through the kidneys and may cause hypokalemia. NOT: Creatinine of 1 mg/dL Normal creatinine levels change with age and gender but are typically between 0.6 and 1.2 mg/dL. New-onset kidney failure with increased creatinine levels may increase the risk of digitalis toxicity. Digoxin of 1.8 ng/mL Normal digoxin levels are 0.5-2 ng/mL. A level of 1.8 ng/mL is in the normal range. Magnesium of 2 mEq/L Normal magnesium levels range from 1.3 to 2.1 mEq/L. Hypomagnesemia may precipitate digitalis toxicity. Furosemide may cause the kidneys to excrete magnesium. Digoxin is a cardiac glycoside medication used to treat the symptoms of heart failure. Depending on the type of heart failure, a client may experience a rapid heart rate that causes poor perfusion. This alteration in perfusion will lead to a backup of fluid in the peripheral tissues and organs. Digoxin improves the ability of the heart to contract, causing the heart rate to slow. Because perfusion is improved, urine output will increase. Extra fluid within the circulation and organs will be removed, causing a reduction in peripheral edema and a reduction in shortness of breath or exertional dyspnea. A nurse cares for a client with heart failure and low blood pressure. After administering prescribed digoxin, which assessment finding does the nurse expect? Decreased heart rate Digoxin is a cardiac glycoside that has a negative chronotropic effect on cardiac cells. This effect causes the heart rate to decrease. NOT: Increased exertional dyspnea Through the improvement of tissue perfusion and diuresis, digoxin will reduce exertional dyspnea. Decreased urine output Digoxin promotes diuresis, increasing urine output. Increased peripheral edema Digoxin promotes tissue perfusion, reducing peripheral edema.

Clostridium difficile (C. diff) and antibiotics

The use of any antibiotic can cause C. difficile, including vancomycin, which is used to treat this condition, but C. difficile is most associated with broad-spectrum antibiotics, especially clindamycin.The destruction of the normal intestinal flora provides an opportunity for C. difficile to grow, causing pseudomembranous colitis. Frequency of C. difficile infections in clients 65 years or older is 10‑fold higher than observed in younger adults. The nurse cares for a group of clients prescribed antibiotic therapy. Which client does the nurse identify as being at the highest risk for developing Clostridium difficile? A 70‑year‑old prescribed clindamycin Clindamycin has one of the highest associations with C. difficile development. This fact, coupled with the client's age, puts this client at the highest risk. NOT: An 80‑year‑old prescribed vancomycin Although the client is of advanced age, vancomycin is rarely associated with C. difficile. This antibiotic is often administered orally for treatment of resistant C. difficile when metronidazole is ineffective. A 24‑year‑old prescribed amoxicillin Amoxicillin is a broad-spectrum antibiotic, which places this client at risk, but the client's age lowers that risk compared to the 70‑year‑old on clindamycin. A 45‑year‑old prescribed co-trimoxazole Co-trimoxazole is a sulfa antibiotic that is only occasionally associated with C. difficile, making this client at lower risk than the other clients. Long-term antibiotic therapy places the client at risk for opportunistic infections, often referred to as superinfections, that can develop due to the destruction of the client's normal bacterial flora. More common infections of this type are Clostridium difficile, which leads to diarrhea, and Candida albicans (yeast infections). Assessing for these infections allows for early detection and treatment. This is especially important in the case of C. difficile, a problematic hospital-acquired infection that can spread to other clients and lead to life-threatening pseudomembranous colitis. A hospitalized client has been prescribed antibiotics for several weeks. Due to the long-term administration of antibiotics, which of the following assessments does the nurse incorporate into the plan of care? Select All That Apply Bowel movements Long-term antibiotics, especially in hospitalized clients, can result in Clostridium difficile due to the destruction of normal gastrointestinal flora. Oral mucosa Antibiotics can increase the risk for Candida albicans, leading to a condition in the oral cavity referred to as thrush. Skin integrity Antibiotics can increase the risk for Candida albicans or yeast infection. This infection will manifest as a rash that often forms in skin folds and areas of increased moisture. NOT: Daily weight Although some antibiotics may affect fluid balance, fluid volume loss or retention is not a typical side effect of long-term antibiotic therapy. If the client develops ongoing diarrhea, body weight might become relevant, but checking daily weight is not indicated in this scenario. Activity tolerance Although the nurse will assess activity tolerance in general in a client being treated for an infection, it is not directly related to the use of antibiotic therapy.

Triamterene/Hydrochlorothiazide (HCTZ)

Triamterene, a potassium-sparing diuretic, reduces fluid volume for clients with congestive heart failure. Potassium-sparing diuretics place clients at risk for hyperkalemia, which may cause lethal arrhythmias. The nurse knows that diarrhea, weakness, and tingling hands are symptoms of hyperkalemia. Additional symptoms of hyperkalemia include palpitations, skipped heart beats, twitching or tingling muscles, muscle weakness, and hyperactive bowel sounds. The nurse immediately notifies the healthcare provider of the bradycardia and then monitors the client closely for additional symptoms of the suspected hyperkalemia. A client who started taking triamterene 3 weeks ago comes into the clinic reporting diarrhea and tingling hands. The nurse notifies the healthcare provider of which assessment finding? Heart rate of 45 beats/min Triamterene, a potassium-sparing diuretic, places the client at risk for hyperkalemia. Cardiac symptoms of hyperkalemia include bradycardia, hypotension, palpitations, and arrhythmias and must be reported immediately. NOT: Temperature of 100.2° F (37.9° C) A temperature of 100.2° F (37.9° C) is above the normal range of 97°-99° F (36.1°-37.2° C). Temperature changes are not associated with hyperkalemia. This is also not considered a fever. The nurse monitors and administers acetaminophen as prescribed. Respirations of 12 breaths/min A respiratory rate of 12 breaths/min is within the normal range of 12-20 breaths/min. Respiratory depression may present with lethal potassium levels. Blood pressure of 98/48 mm Hg A blood pressure of 98/48 mm Hg is within the normal systolic blood pressure range of 90-120 mm Hg. The nurse closely monitors for hypotension in the client with suspected hyperkalemia.

Nitroglycerin (Nitrostat)

Nitroglycerin is a vasodilator that directly affects the smooth muscle of arterial and venous vessels. The relaxation of this muscle causes the vessels to enlarge, improving blood flow to all tissues but primarily to ischemic cardiac tissue. The sympathetic nervous system responds to nitroglycerin by increasing the heart rate. The relaxation of the vessels causes the blood pressure to decrease. Nitroglycerin does not affect the oxygen saturation level, cause palpitations, or increase the respiratory rate. A client with chest pain is given a sublingual nitroglycerin tablet. Which of the following changes in vital signs does the nurse expect? Select All That Apply Increased heart rate Nitroglycerin is a vasodilator, relaxing smooth muscle and promoting blood flow to ischemic areas. Vasodilation causes the body to think that the blood volume has dropped and, subsequently, activates the sympathetic nervous system to increase the heart rate. Decreased blood pressure Nitroglycerin is a vasodilator. The effect on smooth muscle relaxes the arteries and veins and eases the flow of blood through the vessels. This leads to a reduction in blood pressure. NOT: Increased respiratory rate Nitroglycerin does not increase or otherwise affect the rate of respirations. Heart palpitations Nitroglycerin does not cause palpitations. Decreased oxygen saturation level Nitroglycerin does not reduce the oxygen saturation level. The desired effect of nitroglycerin is relief of chest pain by dilating the coronary arteries and reducing cardiac workload by systemic vasodilation. Nitroglycerin dilates systemic veins and arteries, reducing systemic oxygen demands and increasing oxygen-rich blood flow to the heart. A client receives IV nitroglycerin after an acute myocardial infarction. Which client statement indicates to the nurse that the nitroglycerin is having a therapeutic effect? "Can I get up now that I feel better?" The nurse knows the nitroglycerin is having a therapeutic effect when the client reports decreasing chest pain or when anginal symptoms subside. NOT: "I do not feel my heart racing anymore." Beta blockers, diltiazem, and verapamil are associated with decreased heart rate. Nitroglycerin may slightly increase heart rate as a side effect. "Can I have acetaminophen for my headache?" The most common side effect of nitroglycerin is a headache. Presence of a headache does not indicate the nitroglycerin is having a therapeutic effect. "I am feeling a little bit dizzy when I sit up." Postural hypotension is a common side effect of nitroglycerin. The nurse teaches the client to change positions slowly after taking nitroglycerin. Sublingual nitroglycerin, a nitrate, dilates arterial and venous blood vessels, which allows oxygen-rich blood to flow to areas of ischemia. This dilation of blood vessels places the client at risk for hypotension. Nitroglycerin is contraindicated in the presence of hypotension. Another common side effect of nitroglycerin is a headache. The healthcare provider (HCP) prescribes 1 sublingual tablet of nitroglycerin every 5 minutes for a total of 3 doses for a client with chest pain. The client reports 6/10 substernal chest pain 3 minutes after the first dose. What action should the nurse take? Check the client's blood pressure. The nurse monitors the client's blood pressure before, during, and after sublingual nitroglycerin administration, because the client is at risk for hypotension. The nurse notifies the HCP before administering nitroglycerin to a client with hypotension. NOT: Reevaluate the client's pain in 5 minutes. The HCP's prescription is to repeat the dose in 5 minutes if chest pain persists for 3 doses. Waiting another 5 minutes is not following the HCP's order. Notify the HCP of the client's 6/10 pain. The nurse notifies the HCP if the client's chest pain persists after 3 doses or the client becomes too hypotensive to take 3 doses. Place nitroglycerin under the client's tongue. Sublingual means "under the tongue." For proper delivery of sublingual medications, place the medication on the mucosa below the tongue, on the floor of the mouth. This is done every 5 minutes until 3 doses have been delivered, the client reports no chest pain, or the client's blood pressure is too low to continue administering doses.

Sildenafil (Viagra)

Sildenafil (such as Viagra) is a phosphodiesterase inhibitor used to treat impotence. Nitrates, such as nitroglycerin, increase the risk of serious hypotension and should be avoided when taking sildenafil. Insulin, levothyroxine, and acetaminophen do not interact with sildenafil. A client receives a prescription for sildenafil. Which medication does the nurse instruct the client to avoid when taking sildenafil? Nitroglycerin Nitrates, such as nitroglycerin, increase the risk of serious hypotension and should be avoided when taking sildenafil. NOT: Insulin Insulin is a hormone supplement used to treat diabetes mellitus. It does not interact with sildenafil. Levothyroxine Levothyroxine, a thyroid hormone replacement, does not interact with sildenafil. Acetaminophen Acetaminophen, a nonnarcotic analgesic, does not interact with sildenafil.

Tamoxifen (Nolvadex)

Tamoxifen is an estrogen antagonist used in the treatment of breast cancer. It has been shown to increase the risk for thromboembolic events such as pulmonary embolism. Knowing this helps the nurse prioritize looking for evidence of deep vein thrombosis that might otherwise be overlooked during the immediate assessment. Regardless of the reason for dyspnea, oxygen saturation and breath sounds should also be checked. Neither placing the client in the supine position nor administering albuterol (known internationally as salbutamol) are indicated for dyspnea. A client whose only medical diagnosis is breast cancer has been taking tamoxifen as prescribed and develops sudden-onset dyspnea. Which of the following is the nurse's immediate actions? Select All That Apply Assesses oxygen saturation In the case of dyspnea, oxygen saturation is the best indicator if the cause is hypoxia and helps determine the need for supplemental oxygen. Assesses breath sounds Breath sounds can help narrow down the cause of the dyspnea. Inspects the lower legs Because most pulmonary emboli originate from a deep vein thrombosis, the nurse should look for evidence of unilateral edema and redness of the lower legs. NOT: Administers albuterol (salbutamol) The client does not have a diagnosis of asthma or chronic obstructive pulmonary disease, so this medication is not indicated. Places the client in the supine position The client should be placed in a high Fowler position to reduce the work of breathing and improve gas exchange. Tamoxifen is an antineoplastic agent that competes with estrogen for binding sites (also known as an estrogen receptor antagonist). Side effects such as hot flashes and menstrual changes are related to changes in estrogen's ability to bind to receptor sites. Other common side effects related to this effect are edema, mood changes, and nausea. A black box warning on tamoxifen indicates an increased risk for stroke, pulmonary embolism, and uterine malignancies, which have been fatal in some clients. Arrhythmias, hyperglycemia, and ototoxicity are not known to be associated with tamoxifen. A client is prescribed tamoxifen. The nurse lists which of the following side effects as part of client teaching? Select All That Apply Hot flashes Tamoxifen causes vasodilation and flushing that is experienced as hot flashes. Menstrual changes Changes in vaginal discharge and bleeding are common with tamoxifen. NOT: Hyperglycemia Tamoxifen is not known to elevate blood glucose levels. Ototoxicity Tamoxifen is not associated with ototoxicity. Arrhythmias Tamoxifen is not associated with arrhythmias.

Ezetimibe

Ezetimibe is considered a miscellaneous antilipemic medication. It is the only medication that selectively inhibits the absorption of cholesterol in the small intestine and, as a result, the medication will affect total cholesterol, low-density lipoprotein, and triglyceride levels. It has also been shown to improve high-density lipoprotein levels. The effects of this medication are enhanced when taken with a statin, but it can be used alone. Even though an elevated cholesterol level may contribute to the development of hypertension or heart failure, this medication is not prescribed specifically to treat either of these health problems. Ezetimibe has no effect on bone resorption or demineralization and is not used to treat osteoporosis. A client asks why ezetimibe was prescribed. Which health problem does the nurse explain as the reason for this medication? Hyperlipidemia Ezetimibe is a miscellaneous antilipemic medication used to treat elevated cholesterol levels. It is the only medication that selectively inhibits the absorption of cholesterol in the small intestine, reducing the levels of total cholesterol, low-density lipoproteins, and triglycerides. NOT: Hypertension Ezetimibe is a medication that treats elevated cholesterol levels. It has no direct action on blood pressure control. Heart failure Ezetimibe does not treat congestive heart failure. Osteoporosis Ezetimibe has no effect on bone breakdown, which occurs in osteoporosis.

Labetalol (Trandate)

Labetalol is an adrenergic receptor blocker that is selective to α1, β1, and β2 receptors. This results in a decreased heart rate (β1) and an increase in systemic vasodilation (α1), causing a rapid decrease in blood pressure. Orthostatic hypotension and the risk for syncope necessitates placing the client in a supine position and restricting movement until response to treatment is known. The correct order to administer labetalol by repeated intravenous injection is as follows: Place the client in the supine position. Assess blood pressure. Administer the drug over 2 minutes. Wait 5 minutes. Document the client's response.

Metoclopramide (Reglan)

Metoclopramide is a gastrointestinal stimulant that increases the resting tone of the esophageal sphincter to accelerate gastric emptying and intestinal transit. This action facilitates the placement of the feeding tube into the duodenum. The other medications would not make the process easier: Famotidine is an antiulcer medication, loperamide is an antidiarrhea medication, and ondansetron is an antiemetic. A sedated client is to have an enteral feeding tube inserted. Which prescribed medication does the nurse provide to facilitate placing the feeding tube into the duodenum? Metoclopramide Metoclopramide is a gastrointestinal stimulant that increases the resting tone of the esophageal sphincter to accelerate gastric emptying and intestinal transit. This action facilitates the placement of the feeding tube into the duodenum. NOT: Famotidine Famotidine is an antiulcer medication that inhibits histamine in the gastric parietal cells. Loperamide Loperamide is an antidiarrhea medication that inhibits gastrointestinal activity. Ondansetron Ondansetron is an antiemetic that works by blocking serotonin receptors located in the chemoreceptor trigger zone.

Penicillins

Penicillins are one group within the family of β‑lactam antibiotics that also includes cephalosporins, carbapenems, and monobactams. All contain a β‑lactam ring in their biochemical makeup. β‑lactam drugs are among the leading causes of drug-induced anaphylaxis. The client should be made aware of the risk for cross-reactivity and encouraged to wear a medical information bracelet or provide another means of communicating this allergy to healthcare providers. Azithromycin and levofloxacin are considered safe for this client. A client is admitted to the hospital after a severe hypersensitivity reaction to penicillin. Which additional antibiotics does the nurse caution the client regarding use since they may cause an allergic response? Select All That Apply Amoxicillin Amoxicillin is a broad-spectrum β‑lactam antibiotic that can produce an allergic response in clients who have had a serious allergic reaction to penicillins or other β‑lactam antibiotics. Piperacillin-tazobactam Piperacillin is a broad-spectrum β‑lactam antibiotic that can produce an allergic response in clients who have had a serious allergic reaction to penicillins or other β‑lactam antibiotics. Tazobactam is combined with this antibiotic to make it more effective by preventing bacteria from producing an enzyme that resists the action of the piperacillin. This combination is known as Zosyn. Cefazolin Cefazolin is a cephalosporin antibiotic that belongs to the broader class of β-lactam antibiotics. It can produce allergic response in clients who have had a serious allergic reaction to other β-lactam antibiotics, such as penicillin. NOT: Levofloxacin Levofloxacin is a fluoroquinolone antibiotic that does not have cross-sensitivity with β‑lactam antibiotics, so it is considered safe for the client. Azithromycin Azithromycin belongs to the class of macrolide antibiotics that do not have cross-sensitivity with β‑lactam antibiotics, so it is considered safe for the client.

Ciprofloxacin (Cipro)

Photosensitivity is an adverse effect of ciprofloxacin. Teaching should be focused on actions to prevent or limit exposure to the sun, such as using sunscreen when outdoors. Bathing with tepid water and mild soap protects the epidermal layer of the skin but has no effect on photosensitivity. Cornstarch is used to absorb moisture. Applying lotion provides moisture to the skin but has no effect on photosensitivity. A client with osteomyelitis is prescribed ciprofloxacin. Which direction does the nurse provide to ensure the client does not develop an integumentary adverse effect from the medication? "Use sunscreen when outdoors." Photosensitivity is an adverse effect of ciprofloxacin. Teaching should be focused on actions to prevent or limit exposure to the sun, such as using sunscreen when outdoors. NOT: "Apply lotion to the skin several times a day." Applying lotion to the skin several times a day does not help the client taking ciprofloxacin. "Bathe with tepid water and a mild soap." Bathing with tepid water and mild soap does not help the client taking ciprofloxacin. "Apply cornstarch to skin folds." Cornstarch in skin folds helps absorb moisture, but skin moisture is not an expected issue related to ciprofloxacin. Ciprofloxacin is a quinolone antibiotic used to treat Gram-positive and aerobic Gram-negative organisms. This medication has a black box warning because it has been associated with an increased risk of tendonitis and tendon rupture. The client should be instructed about this potential adverse effect. Ciprofloxacin does not adversely affect the respiratory or cardiovascular system. Polyuria is not an adverse effect of this medication. A client is prescribed ciprofloxacin as treatment for a urinary tract infection. About which potential adverse effect does the nurse instruct this client? Tendonitis Ciprofloxacin has a black box warning because it is associated with an increased risk of tendonitis and tendon rupture. The client should be instructed about this potential adverse effect. NOT: Hypotension Ciprofloxacin does not adversely affect the cardiovascular system. Polyuria Polyuria is not identified as an adverse effect of ciprofloxacin. Cough Ciprofloxacin does not adversely affect the respiratory system.

Tretinoin (topical)

Tretinoin is an antiacne medication that works by reversing comedone formation. When used topically, desquamation or peeling, pruritus, and skin irritation occur, which are expected effects of the medication. It is beyond the nurse's scope of practice to discontinue or alter the prescribed dose of a medication. Changing the dose to increase or decrease the amount could adversely affect treatment or cause the development of adverse effects. A client prescribed tretinoin gel reports peeling of the skin on the face. How does the nurse respond to this client? Explains that this is an expected effect Tretinoin is an antiacne medication that works by reversing comedone formation. When used topically, desquamation or peeling, pruritus, and skin irritation occur, which are expected effects of the medication. NOT: Instructs the client to stop the medication The medication should not be discontinued without a healthcare provider's prescription. Teaches the client to increase the frequency of use Increasing the frequency of use could cause the development of adverse effects. Directs the client to reduce the amount used The medication dose should not be changed without a healthcare provider's prescription.

Epinephrine

Ventricular fibrillation, a common rhythm in sudden cardiac arrest, is most successfully treated with prompt defibrillation. Epinephrine (1:10,000 solution), although not shown to improve survival, is the first medication recommended for use during cardiac arrest to improve coronary artery perfusion pressure and the rate of return of spontaneous circulation. Epinephrine causes cardiac stimulation and vasodilation, increasing heart rate and cardiac output. According to the American Heart Association, epinephrine administration should begin following the second shock during resuscitation efforts or after the first shock if an IV is present. The most effective treatment for ventricular fibrillation is rapid defibrillation with a high-energy shock. A client with an established IV site remains in ventricular fibrillation on the cardiac monitor after initial defibrillation with 120 J. Which medication should the nurse initially prepare to administer while cardiopulmonary resuscitation for the client resumes? Epinephrine The client who remains in ventricular fibrillation after initial defibrillation receives 1 mg of epinephrine every 3-5 minutes while ventricular fibrillation persists. If ventricular fibrillation persists after the defibrillation following the initial dose of epinephrine, the nurse may give a 300‑mg bolus of amiodarone. NOT: Atropine Atropine is the initial drug of choice for symptomatic bradycardia according to 2015 American Heart Association guidelines. Atropine increases heart rate and conduction velocity. Dopamine Dopamine infusion treats shock and is the second-line treatment of choice for symptomatic bradycardia according to 2015 American Heart Association guidelines. Amiodarone Amiodarone is administered after a dose of epinephrine and another defibrillation attempt.


Ensembles d'études connexes

MGMT 1105 Organizational Behavior Quiz 8

View Set

Chapter 32: Assessing Older Adults (A & J)

View Set

HTML Quiz Answers-Web Development II

View Set